Skip to main content
Physics LibreTexts

5.5: Static Equilibrium

  • Page ID
    63008
  • \( \newcommand{\vecs}[1]{\overset { \scriptstyle \rightharpoonup} {\mathbf{#1}} } \) \( \newcommand{\vecd}[1]{\overset{-\!-\!\rightharpoonup}{\vphantom{a}\smash {#1}}} \)\(\newcommand{\id}{\mathrm{id}}\) \( \newcommand{\Span}{\mathrm{span}}\) \( \newcommand{\kernel}{\mathrm{null}\,}\) \( \newcommand{\range}{\mathrm{range}\,}\) \( \newcommand{\RealPart}{\mathrm{Re}}\) \( \newcommand{\ImaginaryPart}{\mathrm{Im}}\) \( \newcommand{\Argument}{\mathrm{Arg}}\) \( \newcommand{\norm}[1]{\| #1 \|}\) \( \newcommand{\inner}[2]{\langle #1, #2 \rangle}\) \( \newcommand{\Span}{\mathrm{span}}\) \(\newcommand{\id}{\mathrm{id}}\) \( \newcommand{\Span}{\mathrm{span}}\) \( \newcommand{\kernel}{\mathrm{null}\,}\) \( \newcommand{\range}{\mathrm{range}\,}\) \( \newcommand{\RealPart}{\mathrm{Re}}\) \( \newcommand{\ImaginaryPart}{\mathrm{Im}}\) \( \newcommand{\Argument}{\mathrm{Arg}}\) \( \newcommand{\norm}[1]{\| #1 \|}\) \( \newcommand{\inner}[2]{\langle #1, #2 \rangle}\) \( \newcommand{\Span}{\mathrm{span}}\)\(\newcommand{\AA}{\unicode[.8,0]{x212B}}\)

    Pivots and Torque Reference Points

    The definition of torque (Equation 5.4.6) includes the position vector \(\overrightarrow r\), which points from a reference point to the point where the force is applied. When we are interested in how the torque is accelerating the object rotationally around a fixed point ("pivot"), it is convenient to choose the reference point to be that fixed point. This is because the forces applied at that fixed point (to keep it fixed) provide zero torque when referenced there, and those forces are generally not known. We explore here the effect of changing the reference point in the particular case when there is no net force, though perhaps there could be a net torque. The net torque around a given reference point is:

    \[ \overrightarrow \tau_{net} = \overrightarrow r_1 \times \overrightarrow F_1 + \overrightarrow r_2 \times \overrightarrow F_2 + \dots \]

    The reference point is located at the tails of the \(\overrightarrow r_i\) vectors, but suppose we want to change that reference point. We can do this by simply adding the same constant vector \(\overrightarrow r_o\) to every position vector. This has the effect of shifting the reference point from the point of \(\overrightarrow r_o\) to its tail, as shown in the figure below [Note: The figure shows only two of the many forces applied.]

    Figure 5.5.1 – Changing the Reference Point

    changing_reference_point.png

    The net torque around this new reference point is:

    \[ \begin{array}{l} \overrightarrow \tau_{net} \left(new\right) & = \left(\overrightarrow r_o + \overrightarrow r_1\right) \times \overrightarrow F_1 + \left(\overrightarrow r_o + \overrightarrow r_2\right) \times \overrightarrow F_2 + \dots \\ & = \left[ \overrightarrow r_1 \times \overrightarrow F_1 + \overrightarrow r_2 \times \overrightarrow F_2 + \dots\right] + \overrightarrow r_o \times \left[\overrightarrow F_1 + \overrightarrow F_2 + \dots \right] \\ & = \overrightarrow\tau_{net} \left(original\right) + \overrightarrow r_o \times \overrightarrow F_{net} \end{array} \]

    But we assumed that the net force was zero, so we get the remarkable result that the net torque is the same around every reference point!

    Alert

    As amazing as this result is, be careful not to mistake it for too general of a result. The net torque on an object by a collection of forces is only independent of the reference point if those forces result in zero net force.

    Analyze This

    A board starts at rest and is free of any attachments (it is not pivoted on anything). It is pushed in opposite directions on both of its ends with forces of equal magnitude, at right angles to the board. The forces continue to be applied at right angles with the same magnitude, causing the board to rotate in the manner depicted in the diagram until the board has rotated by \(90^o\).

    rotate_nonuniform_bar.png

    Analysis

    The first thing we notice here is that at all times the board experiences two equal-and-opposite forces. This means that the net force on the board is always zero, and thanks to Newton's 2nd Law, we know that the center of mass of the board cannot accelerate. The board started at rest, so its center of mass in fact never moves. This means that the fixed point on the board (around which the board appears to pivot) is in fact its center of mass.  The mass of the board is therefore not uniformly-distributed, and its red end in the diagram is more dense than the blue end.

    Suppose we only know the length of the board: \(L\). We cannot determine the torques about the center of mass exerted by each force, because we don't know how far from the ends of the board the center of mass is. But we can determine the sum of these torques. From the result derived above, the zero net force allows us to measure the net torque around any point. Choosing one end of the board, the force applied there provides zero torque, and the force on the other end provides a torque of \(\tau=FLsin90^o=FL\). If one wanted to do more work, this same result could be obtained less "cleverly" by calling the distance from one end to the center of mass \(d\), making the distance from the other end to the center of mass \(L-d\). Then multiplying each by \(F\) and adding them together (the torques are in the same direction) gives the same result.

    Static Equilibrium

    We have spent a great deal of time studying motion in all its forms, but now we’re going to step back and look at something called static equilibrium. Simply put, this means unmoving (static), and not about to move (equilibrium). This is a particularly important subject for engineers who aspire to build things that won’t (easily) fall down. From Newton’s laws for linear and rotational motion, we have two conditions for the equilibrium part of this condition:

    • net force on object is zero
    • net torque on object is zero

    We are quite familiar with the net force part of this, but we need to do a bit of work on net torque. We know the formal definition of torque, but there is more we need to understand in order to apply this to static equilibrium problems. The first tool that we can immediately add to our toolbox for solving such problems is the result we got above. If the object is in static equilibrium, then it is experiencing zero net force, which means that no matter what reference point we choose, the net torque will be the same. But the net torque is zero for equilibrium, so we will have the following condition to work with:

    For objects in static equilibrium, the net torque calculated around any reference point whatsoever is zero.

    We will find the flexibility to choose any point we like as a reference to point to be very useful in what is to come.

    Conceptual Question

    For the force diagram below, the force vectors are drawn in the proper locations on the object, and are pointing in the proper directions, but the lengths of the vectors are not to scale. Which of the following statements are true about the effects these forces can have on the motion of this object? Assume that none of the force magnitudes can be set to zero.

    forces_on_blob.png

    1. The force magnitudes can be set so that the object will not accelerate rotationally, while at the same time its center of mass does not accelerate linearly.
    2. There is no way to set the force magnitudes to prevent either linear or rotational acceleration.
    3. The force magnitudes can be set so that either there is no linear acceleration of the object’s center of mass, or there is no rotational acceleration of the object, but both cannot be achieved at the same time.
    4. The force magnitudes can be set so that the object’s center of mass will not accelerate linearly, but there is no way to prevent its rotational acceleration.
    5. The force magnitudes can be set so that the object will not accelerate rotationally, but there is no way to prevent linear acceleration of its center of mass.
    Solution

    (d)
    The two force vectors can be adjusted relative to each other so that their horizontal components cancel. Then both of their magnitudes can be adjusted in the same proportions so that the horizontal net force remains zero, while their combined vertical component of force cancels the other force vector. So zero net force is achievable. However, if we consider a reference point where the middle force acts on the object (giving that middle force zero contribution to torque), the torque of the other two forces will never cancel, no matter what adjustments are made to the force magnitudes. With no way to make the torque vanish, there is no way to prevent rotational acceleration.

    Using Geometry to Determine Torque

    Our definition of torque is all well-and-good, but in practice we rarely define a position vector and take a cross product. Instead, we tend to use the concept behind torque, and then some geometry. The figure below shows two ways to geometrically get to the same torque due to an applied force.

    Figure 5.5.2 – Alternative Methods of Computing Torque

    alternative_torque_calculations.png

    The left version consists of taking only the component of force that is perpendicular to the line joining the reference point and the point where the force is applied, giving the torque magnitude calculation:

    \[ \tau = F_{\bot} d = \left(F\sin\theta\right)d \]

    The right side of the figure shows another useful way to compute the same torque magnitude. Rather than finding the part of \(\overrightarrow F\), it involves finding the perpendicular part of \(\overrightarrow r\). This is done by extending the line of force and then geometrically determining the perpendiular distance from the reference point to that line. The result is the same as above:

    \[ \tau = Fd_{\bot} = F\left(d\sin\theta\right) \]

    The perpendicular distance from the reference point to the line of force is often referred to as the moment-arm, or lever-arm. We will find this to often be the method of choice of computing torques when it comes to solving problems.

    Conceptual Question

    What can you say about the torque applied to the object due to the force \(F\) about the red pivot in the diagram?

    moment_arm_for_crooked_arm.png

    1. it equals \(\frac{1}{2}FL\)
    2. it equals \(\frac{1}{4}FL\)
    3. it is greater than \(\frac{1}{2}FL\)
    4. it is less than \(\frac{1}{4}FL\), but greater than zero
    5. net torques always sum to zero
    Solution

    (c)
    There are a couple of ways to answer this. This first is to extend the force line along \(F\) and look at the perpendicular distance from the pivot to that line (this is the moment arm). It should be clear from the geometry that this moment arm exceeds \(\frac{1}{2}L\), which means the torque must be greater than \(\frac{1}{2}FL\). Another way to see it is to break \(F\) into two separate vectors, one pointing left and the other pointing down. Both of these forces produce clockwise torques, and the horizontal force has a moment arm of \(\frac{1}{2}L\), while the vertical force has a moment arm of \(\frac{3}{4}L\). Since the sum of these two force components exceeds the magnitude of the original force, and since one of them has a moment arm larger than \(\frac{1}{2}L\), then the combined torques must exceed \(\frac{1}{2}FL\).

    Analyze This

    The blob in the figure below is rigid and in static equilibrium. The two forces shown are two of the total of three forces exerted on the object.

    blob_on_grid.png

    Analysis

    The two conditions of equilibrium require that the net force and net torque equal zero. It doesn't take much to find the third force's magnitude and direction from what is given, as it just needs to make a sum that equals zero, which means the third force must be:

    \[\vec F_3=-\left(\vec F_1 + \vec F_2\right)\nonumber\]

    With the two forces at right angles to each other, the magnitude of their sum is:

    \[\left|\vec F_3\right|=\sqrt{F_1^2+F_2^2}\nonumber\]

    We also easily get the angle. Clearly the balancing force \(\vec F_3\) points to-the-left-and-down to cancel these forces. Measuring the angle down from the \(x\)-axis:

    \[\tan\theta = \frac{F_2}{F_1}\nonumber\]

    But of course knowing the force vector is not the whole story. What about where it is applied? Well, clearly it can be applied at many points on the object and still provide the same cancelling torque. Once the force's direction is known, the slope of the "force line" is determined. This line then just needs to be shifted so that the resulting torque cancels the others. And since the net force is already known to be zero, any reference point for this torque can be used.

    Center of Gravity

    Up to now, whenever we have drawn a force diagram of an object, we have always placed the force vector for gravity at its center, while other forces are placed wherever they happen to act on the object. Gravity is somewhat special in that the force actually acts on every single atom in the object, but we can’t draw all of those individual force vectors. Drawing it at the center of mass makes sense from the standpoint of Newton’s second law, since if gravity is the only force, then it accelerates the object, and the part of the object that accelerates is the center of mass.

    Wherever it happens to be appropriate to locate a single gravity force vector on a free-body diagram, it is called the object's center of gravity. We are currently dealing with torque, and the position at which a force acts has become quite important, so we need to examine more closely whether we can declare the center of mass of an object to be its center of gravity.

    We choose as our test subject a horizontal non-uniform rod of length \(L\), and select one of its ends as a reference point. The plan is to add up all of the infinitesimal torques that occur about this reference point due to gravity acting on every particle in the rod, and see if this total torque can be replaced by the entire gravity force acting at a single point (so that we can draw our free-body diagrams with only one gravity force vector!). An arbitrary piece of the rod will be a distance \(x\) from the reference point, and the torque exerted there will be the weight of that piece multiplied by \(x\):

    Figure 5.5.3 – Center of Gravity of a Non-Uniform Rod

    center_of_gravity.png

    \[d\tau = \left(dm\;g\right) x \;\;\; \Rightarrow \;\;\; \tau = \int \limits_0^L dm\;gx = Mg \left[\dfrac{1}{M}\int \limits_0^L dm\;x\right] = Mgx_{cm} \]

    Sure enough, we get the same torque around the reference point if we put a single force vector with magnitude \(Mg\) (the object's full weight) acting at the object's center of mass.

    Alert

    It should be mentioned that there was a rather subtle assumption made in the above discussion – the gravity force is assumed to be the same at all points on the rod. If the gravity force can somehow vary from one end of the rod to the other, then the positions of these two centers will not coincide. If you are wondering how this can ever be the case, the answer is that the scale of distances must be very large, so that there are measurable differences in the gravity force from one point on the object to the other. This will not be an issue for our typically terrestrially-constrained studies, but can arise when talking about orbits of large bodies like moons.

    Note that like center of mass, the center of gravity of an object does not have to lie on the object. For example, a hoop’s center of gravity is located in the empty space at its center. We now know how to locate the position of the gravity force on an object, and locating most other forces will be fairly intuitive (with one notable exception, which we will address next). This will enable us to use torque to analyze a whole range of real-world problems.

    Placement of the Normal Force

    Like the gravity force, the normal force can act at many places at once. When two surfaces come into contact, all of the particles at one surface repel all of the particles at the other. So once again we have the problem of where to draw a single force vector, this time for the normal force. The normal force is different from the gravity force, in one important way – it just compensates for other forces. That is, it adjusts according to other circumstances. Let's use what we know about static equilibrium to see how to place the normal force properly.

    Consider the oddly-shaped object shown in figure below. We'll assume that the object sits at rest on a horizontal surface, the density of this object is not uniform, and that the center of gravity is at the position indicated in the diagram.

    Figure 5.5.4 – Deducing the Normal Force Placement Balancing Only Gravity

    locating normal force.png

    A (rather unsystematic) process for locating the position of the normal force goes like this:

    1. Note that the object is in static equilibrium, which means that the normal force is equal to the weight (net force is zero), and that the net torque around any reference point we care to choose is also zero.
    2. Try various positions for the normal force, and if we can prove that there must be a non-zero net torque around a reference point, then throw that position out.
    3. Repeat step (2) for various positions until one is found that cannot be ruled-out.

    For this object, we could try a normal force vector acting at the center of the base of the object. But then if we choose a reference point between the normal force vector and the weight vector, see see that those two forces must produce a non-zero counter-clockwise torque. We can similarly rule out any position to the right of the weight vector. If we try a position to the left of the weight vector, we get a similar result, this time with the torque being clockwise. We therefore conclude that for this case the normal force vector must be applied exactly where the weight vector intersects the base. No matter where we choose a reference point in that case, the two forces result in equal-and-opposite torques.

    Let's complicate matters some by introducing a second force to our object – suppose we push down on the right side of the object with our thumb, as shown in the figure below.

    Figure 5.5.5a – Deducing the Normal Force Placement Balancing Two Forces

    locating_normal_force.png

    Let's try the same position for the normal force as before – in line with the gravity force. If we choose as a reference a point in line with these two forces, then they create no torque between the two of them, and the added force by the thumb creates a net clockwise torque. This isn't possible for an object in static equilibrium, so the normal force placement has moved from its original placement as a result of the added thumb force. It's easy to see that the normal force hasn't moved left, as placing the reference point at the normal force results in both the weight and the thumb force producing clockwise torques. So the normal force must move right, but how far? Perhaps it moves into line with the thumb force? No... We can choose the reference point to be in line with these two forces (so they both create zero torque), and the gravity force would yield a net counterclockwise torque.

    Figure 5.5.5b – Deducing the Normal Force Placement Balancing Two Forces

    normal_force_placement_labeled.png

    So we conclude that the normal force must act somewhere between the gravity and thumb forces. If we know the magnitudes of these two forces, then we know the magnitude of the normal force (the net force is zero), and in fact we can also determine precisely where a single normal force is acting on the object. Calling the distance between the weight and normal force vector placements \(d_1\) and the distance between the normal force and thumb force vector placements \(d_2\), we can sum the torques around a reference point where the normal force acts (so it contributes no torque) to get:

    \[ 0 = \tau_{net} = -mgd_1 + Fd_2 \;\;\; \Rightarrow \;\;\; \dfrac{d_1}{d_2} = \dfrac{F}{mg} \]

    [Note: In the torque sum, clockwise was chosen as the positive direction.]

    In the diagram, the weight is shown to be greater than the thumb force, making the ratio less than 1, which means the placement of the normal force is closer to the placement of the weight vector than the thumb force vector. If the thumb pushes down more, then the normal force placement moves to the right. Note also that the same result arises if the reference point is chosen elsewhere. For example, if the reference point is chosen to be where the weight force acts, then the net torque equation gives zero contribution from the weight, and contributions from both the normal force (counterclockwise), and the thumb force (clockwise). The normal force can then be written in terms of the weight and thumb force (the net force is zero), giving:

    \[ 0 = \tau_{net} = -Nd_1 + F\left(d_1+d_2\right) \;\;\; \Rightarrow \;\;\; 0 = -\left(mg+\cancel{F}\right)d_1 + \cancel{Fd_1}+Fd_2 \;\;\; \Rightarrow \;\;\; \dfrac{d_1}{d_2} = \dfrac{F}{mg} \]

    Conceptual Question

    Two different blocks are at rest on opposite ends of a smooth uniform wooden plank, which balances at a point that is two-thirds of the length of the plank from one end, as shown in the diagram.  A force of \(F_1\) is applied to the block farthest from the balance point, and a force of \(F_2\) is applied to the other block. Both forces are horizontal and point toward the center of the plank.  As the blocks accelerate due to their respective forces (without friction from the plank), the plank remains balanced. Which of the following can be concluded about the magnitudes of the two forces?

    block_on_plank.png

    1. \(F_1=F_2\)
    2. \(F_1=2F_2\)
    3. \(2F_1=F_2\)
    4. \(F_1>F_2\) (the exact proportion depends upon the mass of the plank)
    5. \(F_1<F_2\) (the exact proportion depends upon the mass of the plank)
    Solution

    (a)
    The system starts at static equilibrium, which means the center of mass of the two blocks + plank system lies directly above the balance point. For the center of mass of this system to not accelerate away from this location, the net force on the system must be zero. This is achieved when the two forces (which are in opposite directions) have equal magnitudes.

    Conceptual Question

    Below is a diagram of a sign hanging from a wall with a boom and a support wire. If the boom is uniform in density (its center of mass represented by the black dot) and weighs about the same as the sign, which of the force vectors shown most closely approximates the direction of the total force on the boom by the wall?

    boom_and_sign.png

    1. A
    2. B
    3. C
    4. D
    5. E
    Solution

    (d)

    The mass of the boom and sign are equal, so their combined center of gravity is three-quarters of the boom length from the wall. We can replace those two weights with a single gravity force acting at that center of gravity. If we choose as our pivot the point of intersection between the line of that gravity force and the tension force, then they both contribute zero torque. The only force left is that of the wall, and for it to not create a net torque around that pivot, it must also pass through that point. It does this if it points in direction D.

    Conditions for Tipping

    Let’s make a slight change to the situation just described. Suppose I push horizontally on the top of the object. What happens to the normal force position as the magnitude of the push increases? Assuming there is a static friction force to prevent the object from sliding, we have a free-body diagram (missing the normal force) that looks like this:

    Figure 5.5.5c – Deducing the Normal Force Placement Balancing Two Forces

    tipping_object.png

    Choosing a pivot point at the intersection point of the gravity and friction forces, we see that the push force exerts a net counterclockwise torque. For the normal force to counteract this (and given that it must push straight up), we find that it must be placed to the left of the center of gravity.

    Let's take a moment to consider the magnitudes of these forces. So long as the object doesn't slide, the static friction force must equal the push. The object doesn't accelerate up or down, so the normal force must have the same magnitude as the gravity force. Both of these conditions are important when we consider what happens when the push force is increased. The friction force also increases until it hits its maximum, at which point the object starts sliding. If we suppose the static friction force doesn't hit its maximum, how is the increased torque by the push compensated by the normal force, if it can't change magnitude? It must move left. But it can only move left for so long, and when it has gone as far as it can go, any added push results in angular acceleration – the object tips.

    Figure 5.5.6 – Tipping

    tipping.png

    Suppose you want to push an object across the floor without tipping it over. To get it to slide, you have to push with a force at least equal to the static friction force, so to avoid tipping, this given amount of force needs to provide as little torque as possible – push at a point close to the bottom. With very little torque from the push force, the normal force can easily remain inside of the edge of the object, and the object won't tip before it slides.

    Stable/Unstable Equilibrium

    If the object is oddly-shaped and the only forces acting on it are gravity and the normal force, then this analysis gives us an answer as to whether the object falls over – if the normal force can be directly beneath the center of gravity, then it will stand up. By “can,” of course we mean that some part of the base that is in contact with the surface (where the normal force acts) must be below the center of gravity.

    In Section 3.7 we discussed stable and unstable equilibrium from the perspective of energy diagrams, and the concept of whether an equilibrium is stable or unstable was first addressed. The idea is that if the system is moved slightly from its equilibrium state, do the forces (or, in our current case, torques) act to return the system to equilibrium (stable), or to continue moving the system away from equilibrium (unstable). How these definitions apply to tipped objects is clear from the free-body diagrams, as shown in the figure below. We can also define a degree of stability to a standing object. We define it as the angle through which we can rotate it such that if we let it go, restoring torques act to return it to its original position.

    Figure 5.5.7 – Stable and Unstable Equilibrium

    stability.png

    Note that this definition of stability matches with what we saw in energy diagrams. Recall that an equilibrium was a point where the potential energy function has zero slope, and the equilibrium is stable if the potential energy grows on both sides of the equilibrium, and is unstable if the potential energy falls off on both sides. Consider what happens to the gravitational potential energy of the object in both cases shown above. In the stable case, tipping the object slightly raises the center of mass of the object (increasing its gravitational potential energy), while in the unstable case a slight tip lowers the center of gravity (decreasing its gravitational potential energy).

    Problem-Solving

    Problems involving static equilibrium can be approached in a very systematic way, the steps of which are outlined below:

    1. Determine the object in static equilibrium you need to analyze and isolate it in a force diagram. This can sometimes be easier said than done. Sometimes the problem involves more than one extended object in contact with each other. In this case, determining the object you choose (or perhaps the combination of both objects) depends upon what you are asked to solve for (usually a force). You can’t really go wrong here, though – if you choose an object that will not give you the answer you need, it should occur to you as you draw the force diagram. Also, you may find that a “wrong” choice of object may simply make your task a bit longer (more simultaneous equations) – annoying, but no real harm done.
    2. Define a linear \(\left(x,y\right)\) coordinate system for force components, and a rotational coordinate system (positive rotation direction) for torques.
    3. Extend each force vector with a dotted line as far as it will go on the page in both directions.
    4. Choose a reference point. For now we won’t worry about choosing a “good” one, choose any – but stick with it for the remaining steps. When you get better at these problems (which you can only achieve by doing them, especially if you do the same problem in multiple ways), you will get better at choosing convenient reference points. Please note that not all static equilibrium problems involve hinges or other “natural” pivots – The reference point doesn't need to be one of these!
    5. Ignoring distractions like the object itself, use geometry to find the perpendicular distance from every force line to the pivot point (i.e. all the moment arms). Do not worry about what angle you use to find these (i.e. it doesn’t have to be the angle from the torque equation \(\tau = rf\sin\theta\)) – just use geometry.
    6. Multiply the moment arm by the magnitude of each force, and this is the magnitude of the torque due to each force.
    7. Determine whether each torque is clockwise or counterclockwise, and give each the appropriate sign when summing the torques and setting that sum equal to zero for the equilibrium torque condition. Note that you could simply alternatively add up all the CW torques, place them on one side of the equation, and set them equal to the sum of the CCW torques on the other side of the equation. This is easier to implement, but loses the “flavor” of what equilibrium is (zero net torque), so I describe it both ways.
    8. If you are lucky (or were clever at the outset), this equation may be all you need in order to find what you are looking for. If it isn’t, you have two alternatives from here...
      • Write out the sum of the forces in the \(x\) and \(y\) directions (or just one of those directions, if that is all you need), and set the net forces equal to zero (another condition for equilibrium). These additional equations should be all you need to find what you are looking for.
      • Choose a new reference point and repeat the torque method described above. Recall that the torques should sum to zero around any point, so this is completely valid. The thing to keep in mind is that wherever you choose your reference point, if a force line goes through it, then that force won’t appear in the torque equation because the moment arm for it is zero. Therefore you can choose your reference point at a spot through which lines for unknown forces pass, eliminating the need to eliminate them using simultaneous equations later. Whatever you do, don’t choose a reference point that lies along a line of the force that you are actually looking for – it doesn’t give you an equation that includes that force!

    What follows is a set of several physical examples of static equilibrium for you to analyze. While they all look quite different, they can all be effectively analyzed in the manner described above. While analysis always goes quite far in solving a problem without even knowing what the question is, this is especially true of these types of problems, so understanding the analysis portion is even more critical for these types of problems than usual.

    Analyze This

    Two painters carry a plank of plywood that they use for scaffolding over their heads on their way to the job site. The plank has a uniform mass distribution. Atop the plank is a can of paint weighing one third as much as the plank. The painter in the rear is holding the plank at the very end and the painter in front is holding the plank one quarter of the the plank length from the front. The can of paint is two-fifths of the plank length from the front. The plank remains horizontal as they carry it.

    painters_carry_plank.png

    Analysis

    We start by identifying the object in equilibrium (the plank), and drawing a free-body diagram for it (we'll call the length of the plank \(L\)). We will choose the pivot to be the back of the plank, and will refer to the weights of the can of paint and plank as W and 3W, respectively. Also we have chosen an \(\left(x,y\right)\) coordinate system and the positive direction of rotation to be clockwise, as shown in the diagram.

    FBD_of_plank.png

    Next apply the conditions of equilibrium. Clearly the \(x\)-direction forces are not meaningful, and the \(y\)-direction force equation and torque equations are:

    \[ \begin{array}{l} vertical\;forces: & & 0 = N_1 -3W -W + N_2 \\ torques: & & 0 = N_1\left(0\right) +3W\left(\frac{1}{2}L\right) +W\left(\frac{3}{5}L\right)-N_2\left(\frac{3}{4}L\right) \end{array} \nonumber \]

    The \(L\)'s cancel out of the torque equation, resulting in a relation between the force exerted by the front painter and the weight of the can:

    \[N_2 = \frac{14}{5}W \nonumber \]

    The total weight carried by the two painters is found from the force equation (or from common sense), and equals \(4W\). So we can compute the percentage of the total load carried by each painter.

    \[ \dfrac{N_2}{4W} = 0.7\nonumber \]

    The front painter carries 70% of the load, and the rear painter 30%.

    Analyze This

    The diagram below depicts a yo-yo on an inclined plane with its string over a massless pulley and attached to a hanging block. The whole system is in static equilibrium. 

    yoyo_on_plane.png

    Analysis

    We start, as always, with a FBD. We are not told about the frictional condition of the surface, so we will leave off a friction force on the yo-yo for now, and see what happens...

    FBD_yoyo_block.png

    Let's take a close look at the FBD of the yo-yo. If we choose its center as a reference point, we see that the gravity force and the normal force don't produce torques, but the tension force does. So this FBD is not correct, as the yo-yo cannot be in static equilibrium with a net torque. There must therefore be a static friction force acting on its outer edge. We even know the direction. About the center of the yo-yo, the torque from the tension is counterclockwise, and to produce a balancing clocwise torque, the static friction force must act down the plane. The new, corrected FBD for the yo-yo is:

    FBD_yoyo.png

    Next we apply the conditions of equilibrium – the sum of the forces and torques add up to zero. Clearly, doing this for the block gives us that the tension equals the weight of the block. The FBD of the yo-yo gives three equations. Calling the radius of the yo-yo's hub \(r\) and the radius of the yo-yo's outer rim \(R\), we have:

    \[\begin{array}{l} F_x: & T-f-Mg\sin\theta = 0 \\ F_y: & N-Mg\cos\theta \\ \tau\;\text{(about center):} & fR-Tr = 0\end{array}\nonumber\]

    With static friction in play, we can also write down the constraint:

    \[f\le\mu_s N\nonumber\]

    As usual, this relation becomes useful if we are told that the system is at some extreme, so that the friction force is maximized. We can now start solving simultaneous equations, but with no knowledge of what we are looking for, we don't know what variables to start eliminating with the algebra, so we will end the anlysis here.


    This page titled 5.5: Static Equilibrium is shared under a CC BY-SA 4.0 license and was authored, remixed, and/or curated by Tom Weideman directly on the LibreTexts platform.

    • Was this article helpful?